\documentclass[11pt]{scrartcl} \usepackage[headheight=1pt, includeheadfoot, headsep=1cm, top=1.5cm, bottom=1.3cm, left=2cm, right=2cm]{geometry} \usepackage{mathtools} \usepackage{amsmath,amsthm, amscd, amssymb, amsfonts} \usepackage[english]{babel} \usepackage{fancyhdr} \usepackage{enumitem} \setlist{itemsep=0.5em} \usepackage{xcolor} \definecolor{forestgreen}{rgb}{0.13, 0.55, 0.13} \usepackage[colorlinks = true, linkcolor = forestgreen, urlcolor = forestgreen, citecolor = forestgreen, anchorcolor = forestgreen]{hyperref} \usepackage{multicol} \usepackage{multienum} \usepackage{euler} \usepackage[OT1]{eulervm} \renewcommand{\rmdefault}{pplx} \usepackage{tikz} \usepackage{mathabx} \usepackage{lastpage} %% header, footer definitions \def\course{MATH 314} \def\courselink{https://www.gsanmarco.com/graph-theory} \def\assignment{HW \#12} \def\assignlink{hw12.tex} \def\due{Due on April 25} \pagestyle{fancy} \fancyhead[L]{\course} \fancyhead[C]{\assignment} \fancyhead[R]{\due} %\renewcommand{\footrulewidth}{0.4pt} %\fancyfoot{} %\fancyfoot[R]{\fontsize{8}{10} \selectfont Page \thepage~of \pageref{LastPage}} %\fancyfoot[L]{\fontsize{10}{12} \selectfont Source file available at \href{\courselink}{\textbf{\courselink}}} %% environments % theorem style \newtheorem{theorem}{Theorem} \newtheorem{lemma}[theorem]{Lemma} \newtheorem{corollary}[theorem]{Corollary} \newtheorem{claim}[theorem]{Claim} \newtheorem{defn}[theorem]{Definition} % definition style \theoremstyle{definition} \newtheorem{problem}{Problem} \newtheorem{Solution}{Solution} % custom \newenvironment{solution}{\noindent\textbf{Solution.}}{\qed} \DeclareMathOperator{\diam}{diam} \newcommand*{\abs}[1]{\lvert #1\rvert} \begin{document} \noindent Unless explicitly requested by a problem, do not include sketches as part of your proof. You are free to use the result from any problem on this (or previous) assignment as a part of your solution to a different problem even if you have not solved the former problem. \vskip10pt \begin{problem}[2pts] Let $G$ be a regular graph. Show that $\chi'(G)=\Delta(G)$ if and only if $G$ is $1$-factorable. \end{problem} \begin{problem}[2pts] \label{perfect} Let $G$ be a bipartite graph with parts $U,W$ where $\abs U=\abs W=k\geq 1$. Prove that if $\abs{E(G)}>k(k-1)$, then $G$ has a perfect matching. Hint: K\"onig's Theorem $\alpha'(G)=\beta(G)$. \end{problem} \begin{problem}[2pts] Let $G$ be a bipartite graph with parts $U,W$ and fix an integer $k\geq 1$. Let $U=U_1\sqcup\dots\sqcup U_k$ and $W=W_1\sqcup\dots\sqcup W_k$ be any partitions (some of the $U_i$'s or $W_j$'s may be empty). Note that $\abs U$ and $\abs W$ have nothing to do with $k$; $k$ is just the number of pieces in each partition. Prove that if $\abs{E(G)}